The simple function are dense in $L^{infty} (X)$ if $mu (X) lt infty$












0












$begingroup$



Let $(X,M,mu)$ be a measure space, show that:

The simple function are dense in $L^{infty} (X)$ if $mu (X) lt infty$




This is an exercise in Stein's functional analysis Page 35.

If $f in L^infty$ , then for any $e gt0$ there exist $N$ , $f lt Ne$ , a.e..

So use $E_n={x:-Ne +ne le f lt -Ne +(n+1)e } $ and let $$f_n(x)=sum_{n=0}^{2N} (-Ne+ne)X_{E_n}(x)$$ where $X_{E_n}$ is the character function of $E_n$ , $0 le n le 2N$, then I can get $$vert vert f-f_n vert vert lt e$$
Since I didn't use the assumption that $mu (X) lt infty$ . Did I make something wrong in the proof ?










share|cite|improve this question











$endgroup$








  • 1




    $begingroup$
    I don't have the book at hand to check for myself, but you might look closely at their definition of simple function. It may require that each set $E_n$ has finite measure.
    $endgroup$
    – Umberto P.
    Dec 4 '18 at 17:38










  • $begingroup$
    Thank you ! I get the point now.
    $endgroup$
    – J.Guo
    Dec 4 '18 at 17:43










  • $begingroup$
    Nope. Generally it is not required for simple functions to have finite (in measure) support. However you proof is wrong. But just slightly. You have notation abuse. Your $f_{n}$ does not depend on $n$. It depends only $N$. So this proof can't be accepted as your choice of $N$ depends on fixed $e$. (However this is right way to think. Soon you'll get the proof. I think so.)
    $endgroup$
    – kolobokish
    Dec 4 '18 at 18:13
















0












$begingroup$



Let $(X,M,mu)$ be a measure space, show that:

The simple function are dense in $L^{infty} (X)$ if $mu (X) lt infty$




This is an exercise in Stein's functional analysis Page 35.

If $f in L^infty$ , then for any $e gt0$ there exist $N$ , $f lt Ne$ , a.e..

So use $E_n={x:-Ne +ne le f lt -Ne +(n+1)e } $ and let $$f_n(x)=sum_{n=0}^{2N} (-Ne+ne)X_{E_n}(x)$$ where $X_{E_n}$ is the character function of $E_n$ , $0 le n le 2N$, then I can get $$vert vert f-f_n vert vert lt e$$
Since I didn't use the assumption that $mu (X) lt infty$ . Did I make something wrong in the proof ?










share|cite|improve this question











$endgroup$








  • 1




    $begingroup$
    I don't have the book at hand to check for myself, but you might look closely at their definition of simple function. It may require that each set $E_n$ has finite measure.
    $endgroup$
    – Umberto P.
    Dec 4 '18 at 17:38










  • $begingroup$
    Thank you ! I get the point now.
    $endgroup$
    – J.Guo
    Dec 4 '18 at 17:43










  • $begingroup$
    Nope. Generally it is not required for simple functions to have finite (in measure) support. However you proof is wrong. But just slightly. You have notation abuse. Your $f_{n}$ does not depend on $n$. It depends only $N$. So this proof can't be accepted as your choice of $N$ depends on fixed $e$. (However this is right way to think. Soon you'll get the proof. I think so.)
    $endgroup$
    – kolobokish
    Dec 4 '18 at 18:13














0












0








0





$begingroup$



Let $(X,M,mu)$ be a measure space, show that:

The simple function are dense in $L^{infty} (X)$ if $mu (X) lt infty$




This is an exercise in Stein's functional analysis Page 35.

If $f in L^infty$ , then for any $e gt0$ there exist $N$ , $f lt Ne$ , a.e..

So use $E_n={x:-Ne +ne le f lt -Ne +(n+1)e } $ and let $$f_n(x)=sum_{n=0}^{2N} (-Ne+ne)X_{E_n}(x)$$ where $X_{E_n}$ is the character function of $E_n$ , $0 le n le 2N$, then I can get $$vert vert f-f_n vert vert lt e$$
Since I didn't use the assumption that $mu (X) lt infty$ . Did I make something wrong in the proof ?










share|cite|improve this question











$endgroup$





Let $(X,M,mu)$ be a measure space, show that:

The simple function are dense in $L^{infty} (X)$ if $mu (X) lt infty$




This is an exercise in Stein's functional analysis Page 35.

If $f in L^infty$ , then for any $e gt0$ there exist $N$ , $f lt Ne$ , a.e..

So use $E_n={x:-Ne +ne le f lt -Ne +(n+1)e } $ and let $$f_n(x)=sum_{n=0}^{2N} (-Ne+ne)X_{E_n}(x)$$ where $X_{E_n}$ is the character function of $E_n$ , $0 le n le 2N$, then I can get $$vert vert f-f_n vert vert lt e$$
Since I didn't use the assumption that $mu (X) lt infty$ . Did I make something wrong in the proof ?







real-analysis functional-analysis






share|cite|improve this question















share|cite|improve this question













share|cite|improve this question




share|cite|improve this question








edited Dec 4 '18 at 17:32







J.Guo

















asked Dec 4 '18 at 17:20









J.GuoJ.Guo

3229




3229








  • 1




    $begingroup$
    I don't have the book at hand to check for myself, but you might look closely at their definition of simple function. It may require that each set $E_n$ has finite measure.
    $endgroup$
    – Umberto P.
    Dec 4 '18 at 17:38










  • $begingroup$
    Thank you ! I get the point now.
    $endgroup$
    – J.Guo
    Dec 4 '18 at 17:43










  • $begingroup$
    Nope. Generally it is not required for simple functions to have finite (in measure) support. However you proof is wrong. But just slightly. You have notation abuse. Your $f_{n}$ does not depend on $n$. It depends only $N$. So this proof can't be accepted as your choice of $N$ depends on fixed $e$. (However this is right way to think. Soon you'll get the proof. I think so.)
    $endgroup$
    – kolobokish
    Dec 4 '18 at 18:13














  • 1




    $begingroup$
    I don't have the book at hand to check for myself, but you might look closely at their definition of simple function. It may require that each set $E_n$ has finite measure.
    $endgroup$
    – Umberto P.
    Dec 4 '18 at 17:38










  • $begingroup$
    Thank you ! I get the point now.
    $endgroup$
    – J.Guo
    Dec 4 '18 at 17:43










  • $begingroup$
    Nope. Generally it is not required for simple functions to have finite (in measure) support. However you proof is wrong. But just slightly. You have notation abuse. Your $f_{n}$ does not depend on $n$. It depends only $N$. So this proof can't be accepted as your choice of $N$ depends on fixed $e$. (However this is right way to think. Soon you'll get the proof. I think so.)
    $endgroup$
    – kolobokish
    Dec 4 '18 at 18:13








1




1




$begingroup$
I don't have the book at hand to check for myself, but you might look closely at their definition of simple function. It may require that each set $E_n$ has finite measure.
$endgroup$
– Umberto P.
Dec 4 '18 at 17:38




$begingroup$
I don't have the book at hand to check for myself, but you might look closely at their definition of simple function. It may require that each set $E_n$ has finite measure.
$endgroup$
– Umberto P.
Dec 4 '18 at 17:38












$begingroup$
Thank you ! I get the point now.
$endgroup$
– J.Guo
Dec 4 '18 at 17:43




$begingroup$
Thank you ! I get the point now.
$endgroup$
– J.Guo
Dec 4 '18 at 17:43












$begingroup$
Nope. Generally it is not required for simple functions to have finite (in measure) support. However you proof is wrong. But just slightly. You have notation abuse. Your $f_{n}$ does not depend on $n$. It depends only $N$. So this proof can't be accepted as your choice of $N$ depends on fixed $e$. (However this is right way to think. Soon you'll get the proof. I think so.)
$endgroup$
– kolobokish
Dec 4 '18 at 18:13




$begingroup$
Nope. Generally it is not required for simple functions to have finite (in measure) support. However you proof is wrong. But just slightly. You have notation abuse. Your $f_{n}$ does not depend on $n$. It depends only $N$. So this proof can't be accepted as your choice of $N$ depends on fixed $e$. (However this is right way to think. Soon you'll get the proof. I think so.)
$endgroup$
– kolobokish
Dec 4 '18 at 18:13










1 Answer
1






active

oldest

votes


















0












$begingroup$

It is true that the simple functions that are not necessarily supported on sets of finite measure (simple functions are just measurable functions on $X$ with a finite image) are dense in $L_{infty}(mu)$ regardless of whether $mu$ is finite. Note that you showed it for none negative bounded functions (I think), but it is easily extensible to complex functions in $L_{infty}(mu)$.






share|cite|improve this answer









$endgroup$













    Your Answer





    StackExchange.ifUsing("editor", function () {
    return StackExchange.using("mathjaxEditing", function () {
    StackExchange.MarkdownEditor.creationCallbacks.add(function (editor, postfix) {
    StackExchange.mathjaxEditing.prepareWmdForMathJax(editor, postfix, [["$", "$"], ["\\(","\\)"]]);
    });
    });
    }, "mathjax-editing");

    StackExchange.ready(function() {
    var channelOptions = {
    tags: "".split(" "),
    id: "69"
    };
    initTagRenderer("".split(" "), "".split(" "), channelOptions);

    StackExchange.using("externalEditor", function() {
    // Have to fire editor after snippets, if snippets enabled
    if (StackExchange.settings.snippets.snippetsEnabled) {
    StackExchange.using("snippets", function() {
    createEditor();
    });
    }
    else {
    createEditor();
    }
    });

    function createEditor() {
    StackExchange.prepareEditor({
    heartbeatType: 'answer',
    autoActivateHeartbeat: false,
    convertImagesToLinks: true,
    noModals: true,
    showLowRepImageUploadWarning: true,
    reputationToPostImages: 10,
    bindNavPrevention: true,
    postfix: "",
    imageUploader: {
    brandingHtml: "Powered by u003ca class="icon-imgur-white" href="https://imgur.com/"u003eu003c/au003e",
    contentPolicyHtml: "User contributions licensed under u003ca href="https://creativecommons.org/licenses/by-sa/3.0/"u003ecc by-sa 3.0 with attribution requiredu003c/au003e u003ca href="https://stackoverflow.com/legal/content-policy"u003e(content policy)u003c/au003e",
    allowUrls: true
    },
    noCode: true, onDemand: true,
    discardSelector: ".discard-answer"
    ,immediatelyShowMarkdownHelp:true
    });


    }
    });














    draft saved

    draft discarded


















    StackExchange.ready(
    function () {
    StackExchange.openid.initPostLogin('.new-post-login', 'https%3a%2f%2fmath.stackexchange.com%2fquestions%2f3025860%2fthe-simple-function-are-dense-in-l-infty-x-if-mu-x-lt-infty%23new-answer', 'question_page');
    }
    );

    Post as a guest















    Required, but never shown

























    1 Answer
    1






    active

    oldest

    votes








    1 Answer
    1






    active

    oldest

    votes









    active

    oldest

    votes






    active

    oldest

    votes









    0












    $begingroup$

    It is true that the simple functions that are not necessarily supported on sets of finite measure (simple functions are just measurable functions on $X$ with a finite image) are dense in $L_{infty}(mu)$ regardless of whether $mu$ is finite. Note that you showed it for none negative bounded functions (I think), but it is easily extensible to complex functions in $L_{infty}(mu)$.






    share|cite|improve this answer









    $endgroup$


















      0












      $begingroup$

      It is true that the simple functions that are not necessarily supported on sets of finite measure (simple functions are just measurable functions on $X$ with a finite image) are dense in $L_{infty}(mu)$ regardless of whether $mu$ is finite. Note that you showed it for none negative bounded functions (I think), but it is easily extensible to complex functions in $L_{infty}(mu)$.






      share|cite|improve this answer









      $endgroup$
















        0












        0








        0





        $begingroup$

        It is true that the simple functions that are not necessarily supported on sets of finite measure (simple functions are just measurable functions on $X$ with a finite image) are dense in $L_{infty}(mu)$ regardless of whether $mu$ is finite. Note that you showed it for none negative bounded functions (I think), but it is easily extensible to complex functions in $L_{infty}(mu)$.






        share|cite|improve this answer









        $endgroup$



        It is true that the simple functions that are not necessarily supported on sets of finite measure (simple functions are just measurable functions on $X$ with a finite image) are dense in $L_{infty}(mu)$ regardless of whether $mu$ is finite. Note that you showed it for none negative bounded functions (I think), but it is easily extensible to complex functions in $L_{infty}(mu)$.







        share|cite|improve this answer












        share|cite|improve this answer



        share|cite|improve this answer










        answered Dec 4 '18 at 17:48









        pitariverpitariver

        317112




        317112






























            draft saved

            draft discarded




















































            Thanks for contributing an answer to Mathematics Stack Exchange!


            • Please be sure to answer the question. Provide details and share your research!

            But avoid



            • Asking for help, clarification, or responding to other answers.

            • Making statements based on opinion; back them up with references or personal experience.


            Use MathJax to format equations. MathJax reference.


            To learn more, see our tips on writing great answers.




            draft saved


            draft discarded














            StackExchange.ready(
            function () {
            StackExchange.openid.initPostLogin('.new-post-login', 'https%3a%2f%2fmath.stackexchange.com%2fquestions%2f3025860%2fthe-simple-function-are-dense-in-l-infty-x-if-mu-x-lt-infty%23new-answer', 'question_page');
            }
            );

            Post as a guest















            Required, but never shown





















































            Required, but never shown














            Required, but never shown












            Required, but never shown







            Required, but never shown

































            Required, but never shown














            Required, but never shown












            Required, but never shown







            Required, but never shown







            Popular posts from this blog

            Plaza Victoria

            In PowerPoint, is there a keyboard shortcut for bulleted / numbered list?

            How to put 3 figures in Latex with 2 figures side by side and 1 below these side by side images but in...